- Wed Jun 27, 2018 4:23 pm
#47091
Complete Question Explanation
(The complete setup for this game can be found here: lsat/viewtopic.php?t=11700)
The correct answer choice is (B)
The question stem eliminates two services—G and S—from being scheduled for the sixth day. As shown on the final diagram for the game, only G, L, and S can be scheduled for the sixth day, so when G and S are removed from consideration, we can conclude that L must be scheduled for the sixth day. Given that this is a Must Be True question, you should immediately seek this statement among the answer choices. Answer choice (B) states that inference, and answer choice (B) is correct.
Answer choice (A): This answer choice could be true, but it does not have to be true, and so it is incorrect. Note how question #4, answer choice (A) can be used to disprove this answer choice (A is actually the only answer from question #4 that can be used because neither G nor S is scheduled for the sixth day).
Answer choice (B): This is the correct answer choice.
Answer choice (C): This answer choice could be true, but it does not have to be true, and so it is incorrect.
Answer choice (D): This answer choice cannot be true (there would not be enough room to schedule both G and S), and so it is incorrect.
Answer choice (E): This answer choice could be true, but it does not have to be true, and so it is incorrect.
(The complete setup for this game can be found here: lsat/viewtopic.php?t=11700)
The correct answer choice is (B)
The question stem eliminates two services—G and S—from being scheduled for the sixth day. As shown on the final diagram for the game, only G, L, and S can be scheduled for the sixth day, so when G and S are removed from consideration, we can conclude that L must be scheduled for the sixth day. Given that this is a Must Be True question, you should immediately seek this statement among the answer choices. Answer choice (B) states that inference, and answer choice (B) is correct.
Answer choice (A): This answer choice could be true, but it does not have to be true, and so it is incorrect. Note how question #4, answer choice (A) can be used to disprove this answer choice (A is actually the only answer from question #4 that can be used because neither G nor S is scheduled for the sixth day).
Answer choice (B): This is the correct answer choice.
Answer choice (C): This answer choice could be true, but it does not have to be true, and so it is incorrect.
Answer choice (D): This answer choice cannot be true (there would not be enough room to schedule both G and S), and so it is incorrect.
Answer choice (E): This answer choice could be true, but it does not have to be true, and so it is incorrect.